Ajastako muka ei saa otetta?

Aloittaja Joksa, 18.09.2013, 18:22:39

« edellinen - seuraava »

Eusa

Yleisen sunhteellisuusteorian (erään marginaaliapproksimaation) mustassa aukossa jo olevan kappaleen sisäisissä vuorovaikutuksissa ajatellaan kaikkien signaalien liikkuvan horisontin suhteen sisäänpäin, myös ulospäin "horisonttia kohti" ammutun säteilyn. Se, että onko tämä esim. kvanttimekaanisesti mahdollista saada sisäisesti ristiriidattomaksi koskaan, on eri kysymys ja saattaa jäädä turhaksi puntaroinniksi, jos mustat aukot todetaan olemattomiksi.

mistral

Vaikeita kysymyksiä. Otin Wikistä tämän:

Musta aukko syntyy raskaimmista tähdistä supernovaräjähdyksessä. Kappale luhistuu mustaksi aukoksi, jos sen säde alittaa Schwarzschildin säteenä tunnetun matkan. Tämän etäisyyden sisäpuolella aika-avaruus on niin vahvasti kaareutunut, että jokainen valonsäde pyrkii kohti keskustaa. Koska aukosta pakeneminen vaatisi valoa nopeamman liikkeen, kaikki materia Schwarzschildin säteen sisäpuolella luhistuu keskustaa kohti.

Schwarzschildin säde voidaan laskea kaavasta:

    r s = 2 G M c 2 {\displaystyle r_{s}={2GM \over c^{2}}} {\displaystyle r_{s}={2GM \over c^{2}}},

jossa G on gravitaatiovakio ( 6 , 67 × 10 − 11 {\displaystyle 6{,}67\times 10^{-11}} {\displaystyle 6{,}67\times 10^{-11}} Nm2/kg2), M on kohteen massa ja c on valon nopeus. Massaltaan Maan kokoisen kappaleen Schwarzschildin säde on noin 9 mm.

Itse asiassa S:n säde näyttäisi olevan määritelty massallisille hiukkasille! Siis ei niinkään fotoneille. Ja jos fotonit sittenkin pääsee nousemaan horisontin alapuolelta, niin kumpi määrittää ajankulun:  1) massallinen hiukkanen vai 2) massaton hiukkanen (fotoni)?

Mutta ensin pitäisi ratkaista mysteeri fotonin nousemisesta horisontin alapuolelta. Yritän selittää mitä tarkoitan. Jos oletetaan neutronitähdellä olevan g-kaivon syvyys 0,5x mustan aukon kaivon horisontin syvyydestä ja sieltä nousee suoraan zeniittiin 0,001mm fotoni, niin eikö perinteisesti olla laskettu että 0,5x syvyydellä aika kuluu 2x hitaammin kuin Maassa ja kun fotoni nousee, pitenisi sen aalto 0,002mm:iin.

Jos fotoni nyt laitetaan horisontin pintaan ja annetaan sille etapiksi nousta ensin neutronitähden kaivon syvyydelle ja sitten jatkaa eteenpäin vapaaseen avaruuteen, menisi minun järkeni mukaan näin: 0,001mm nousee n.tähden kenttään ja menettää puolet massastaan (0,002mm) ja nousee vapauteen menettäen puolet massastaan (0,004mm).

Edelleen, jos fotoni lähtee horisontin alapuolelta, se lähtisi 0,001mm ja tulisi horisonttiin 0,002mm:nä ja nousisi n.tähden syvyydelle 0,004mm:nä ja vapaaseen avaruuteen 0,008mm:nä.

Eli idea näyttää hämmentävältä mutta perustuu siihen ettei fotoni käyttäydy kuin massallinen hiukkanen. Massallinen hiukkanen menettää noustessaan jatkuvasti nopeutta pysähtymiseen asti, kun taas fotoni ei. Se vaan puolittuu ja puolittuu kymmeniä kertoja. Tai mihin sen rajan vetää, voihan mielivaltaisesti asettaa rajan 3/4 osaan. Näin sen energia laskisi 3/4 osan kerrallaan, sama mihin raja vedetään, jakautuminen tapahtuisi kymmeniä tai satoja kertoja.

Eli kun Schwarzschild teki kaavan pakonopeudesta, niin kuinka se oikeasti toimii fotoneille?

lisäys:
Näköjään S:n kaava ei kopioitunut vaan meni sillisalaatiksi...

Kaizu

Fotoni menettää energiaa niin että taajuus pienenee = aallonpituus kasvaa. Nopeus on aina sama eli valon nopeus.

Kaizu
Kai Forssen

mistral

Lainaus käyttäjältä: Kaizu - 07.03.2017, 13:57:51
Fotoni menettää energiaa niin että taajuus pienenee = aallonpituus kasvaa. Nopeus on aina sama eli valon nopeus.

Kaizu

Näinhän se on, ja siksi en ymmärrä että pakonopeus mittapuuta sovelletaan hiukkaselle jonka nopeus ei muutu. Käytin jossain viestissä rakettia vertauskuvana fotonista. Jos siis oletetaan että raketti nousee tasaista vauhtia avaruuteen, kevenee se koko ajan polttoaineen palaessa. Jos raketti on kartion muotoinen, pienenee sen tulipesä koko ajan koska kartion palaessa se ohenee, näin polttoaineen kulutus pienenee. Vastapainoksi sen massakin pienenee jolloin nopeus pysyy koko ajan samana. Lopulta kartio on millin ohut kärki, mutta sama vauhti on edelleen päällä. Millinkin voi jakaa osiin jolloin matka vaan jatkuu.
Näin ihmettelen kuinka valo ei pääsisi horisontin alta pakoon, tietysti sillä edellytyksellä että se nousee suoraan zeniittiin. Jos se kallistuu sivuun ja kaartaa 90 asteen kulmaan ennen 1,5x Schwarzschildin sädettä, syöksyy se takaisin mustaan aukkoon.

pnuu

Lainaus käyttäjältä: mistral - 07.03.2017, 18:44:13
Näin ihmettelen kuinka valo ei pääsisi horisontin alta pakoon, tietysti sillä edellytyksellä että se nousee suoraan zeniittiin. Jos se kallistuu sivuun ja kaartaa 90 asteen kulmaan ennen 1,5x Schwarzschildin sädettä, syöksyy se takaisin mustaan aukkoon.

Mustan aukon luona avaruus kaareutuu niin paljon, että "suoraan ylös" ei ole "suoraan ylös", vaan avaruuden käpertyessä itsensä ympäri tuollaista suuntaa ei enää ole. Valo etenee suoraan, joka tässä tapauksessa on ulkoa päin katsoen melkoinen sykkyrä.
Panu Lahtinen
"You haven't really been anywhere until you've got back home",
Twoflower in "The Light Fantastic"

mistral

Lainaus käyttäjältä: pnuu - 07.03.2017, 20:04:33
Mustan aukon luona avaruus kaareutuu niin paljon, että "suoraan ylös" ei ole "suoraan ylös", vaan avaruuden käpertyessä itsensä ympäri tuollaista suuntaa ei enää ole. Valo etenee suoraan, joka tässä tapauksessa on ulkoa päin katsoen melkoinen sykkyrä.

Niin, tuohan on perinteinen ajattelutapa. Mutta jos rakettivertausta ajattelee, niin siinä näyttäisi fotonin nousukyky olevan ihan eri luokkaa kuin pakonopeus ajattelun perusteella. Annan vielä lisävinkin:

Kun yleisesti sanotaan että valo voi paeta horisontin ulkopuolelta, tämä klassinen käsitys. No nyt voidaan kysyä, riittääkö fotonille mikä tahansa aallonpituus jotta se voi paeta. Käsittääkseni vastaus on että riittää, ei siis aseteta ehdoksi että sen pitää olla röntgen-, gamma- tai vieläkin lyhytaaltoisempaa, vaan riittää että se on vaikkapa 500nm valoa. Tässä on kummallisuus. Nimittäin jos tämä 500nm:n fotoni saa kaverikseen horisontin alapuolelta tulevan toisen fotonin joka on kanssa 500nm, niin teorian mukaan kaveri on tuhoon tuomittu mutta ulkopuolelta starttaava pääsee vapauteen. Vaikka molemmat menisivät suoraan ylöspäin. Tämä "tuhoon tuomittu" olisi ollut sisäpuolella vaikkapa 250nm fotoni startatessaan jossain tosi syvällä. Elikkä klassinen teoria näyttää epäloogiselta. Tai sitten geodeesit jostain kumman syystä kaartaa niin paljon että kaikki fotonit joutuu alottamaan kaartamisen heti alkumetreillä, en vaan ymmärrä miksi niin olisi. Tai voihan kaavoissa olla jokin selitys, en tiedä.

Kuitenkin tulee mieleen kysymys, mistä tiedetään että ns. horisontissa aika pysähtyy. Jospa ajan kulu on sidottu fotonin värähtelytaajuuteen tai sen käänteislukuun, miten sen sanoisi. Ja jos singulariteetti on sittenkin olemassa ja vasta siellä aika pysähtyy. Tämä romuttaisi perinteisen käsityksen siitä että Maasta katsoen horisontti olisi jäätynyt (koska aika olisi siellä pysähtynyt). Gravitaatiokaivon aikadilataatiota ei voi testata, siis että pitääkö paikkansa ajan pysähtyminen horisontissa, mitä jos ei pidäkään?

mistral

Lainaus käyttäjältä: Joksa - 07.03.2017, 21:23:12

Jotta tapahtumahorisonti olisi pinta jonka takaa ei saada mitään informaatiota niin sen tasossa pysyttelyä varten tarvittavan nousutehon pitäisi olla kappaleelle valon nopeuden antava teho, ja tämä on eri vaatimus kuin pakonopeus.

Joo, ei tapahtumahorisontissa ole mitään "rajamerkkejä" tai lasikattoa jonka kohdalla luonnonlait muuttuisivat, kyllä se voidaan läpäistä sekä sisään että ulospäin. Tosin massallisten hiukkasten tai kappaleiden kohdalla on saavutettu valon nopeus, ainakin niitten kohdalla jotka on saaneet syöksyä ilman jarruttavia kolareita. Tämä on täysin epätavallista, pieni hiekanmurunen olisi massaenergialtaan painavampi kuin Jupiter. Tässä mielessä horisontti on erikoinen paikka, mutta onko siellä aika pysähtynyt, on arvoitus.

jussi_k_kojootti


Tein tämmöisen pienen laskukoneen, jolla voi tarkastella fotonin punasiirtymää Schwarzschildin aukkojen läheisyydessä.  Tämä lähti siitä, kun halusin kertoa Mistralille jotain "punasiirtymän puoliintumisten lopputtomasta sarjasta", tai jostain semmoisesta (tai, ei ollenkaan semmoisesta ;-)).

Pari huomiota joita ei vielä tuolla selitysosioissa ole -- negatiivinen z tarkoittaa sinisiirtymää, siis että emitteri on kauempana horisontista kuin havaitsija.  Laskurin "käyttökelpoinen alue" ei ulotu tilanteisiin, joissa etäisyydet ~< 1e-52 -- eli protonin horisonttia ei pysty laskemaan, mutta solun vielä pystyy.

http://rubor.org/schwarzschildarb.php





jussi kantola / oulun arktos
CG-5 GOTO + KWIQ-guiding + SW80ED  // 10" dobson // canon eos 450d mod & 400d / ASI 120MM
http://astrobin.com/users/jussi_k_kojootti/
http://oulunarktos.fi/

mistral

Lainaus käyttäjältä: Joksa - 08.03.2017, 15:40:24
Wikipedian pintagravitaatiota käsittelvässä osassa löytyy mielenkiintoista tietoa myös tapahtumahorisonttien osalta:  https://fi.wikipedia.org/wiki/Pintagravitaatio#Mustan_aukon_pintagravitaatio

Schwarzschildin määritelmän mukainen tapahtumahorisontti ole mitään läpi päästämätön Killingin horisontti, sellainen olisi ilmeisesti vasta sellainen staattinen Killingin horisontti jonka pintagravitaatio olisi ääretön.

Näköjään suhteellisuusteoria ei tuota horisonttiin todellista g-kiihtyvyysarvoa:

"Mustien aukkojen tapauksessa pintagravitaatiota ei voida määritellä kiihtyvyydeksi, jonka testikappale saisi sellaisen pinnalla. Tämä johtuu siitä, että tapahtumahorisontilla testikappaleen kiihtyvyys olisi suhteellisuusteorian mukaan ääretön. Tämän vuoksi käytetään renormalisoitua arvoa, joka vastaa newtonilaista arvoa heikkojen gravitaatiokenttien tapauksessa, joita voidaan käsitellä Newtonin fysiikan avulla"

On outoa ajatella että kiihtyvyys menisi äärettömään jo Schwarzschildin säteen kohdalla...täytyy mietiskellä vielä.

mistral

Lainaus käyttäjältä: ketarax - 10.03.2017, 11:45:01
Tein tämmöisen pienen laskukoneen, jolla voi tarkastella fotonin punasiirtymää Schwarzschildin aukkojen läheisyydessä.  Tämä lähti siitä, kun halusin kertoa Mistralille jotain "punasiirtymän puoliintumisten lopputtomasta sarjasta", tai jostain semmoisesta (tai, ei ollenkaan semmoisesta ;-)).

Pari huomiota joita ei vielä tuolla selitysosioissa ole -- negatiivinen z tarkoittaa sinisiirtymää, siis että emitteri on kauempana horisontista kuin havaitsija.  Laskurin "käyttökelpoinen alue" ei ulotu tilanteisiin, joissa etäisyydet ~< 1e-52 -- eli protonin horisonttia ei pysty laskemaan, mutta solun vielä pystyy.

http://rubor.org/schwarzschildarb.php

Kopioin alemman taulukon,
  R1 observer distance=variable,   f1=observer frequency,   z=redshift,  ja lisäksi säteilijän korkeus 1x10^-3m= 1mm horisontin yläpuolella

R1 [m]          f1 [Hz]        z
1.000000e+0   3.162812e+18   30.62
1.000000e+1   1.001691e+18   98.83
1.000000e+2   3.215354e+17   310.01
1.000000e+3   1.156937e+17   863.35
1.000000e+4   6.621960e+16   1509.13
1.000000e+5   5.903419e+16   1692.93
1.000000e+6   5.826693e+16   1715.24
1.000000e+7   5.818965e+16   1717.52
1.000000e+8   5.818192e+16   1717.75
1.000000e+9   5.818114e+16   1717.77

Luetaanko ylin rivi näin: havaitsija on 1m korkeudella ja mittaa horisontista tulevan taajuuden 3,16x10^18Hz. Punasiirtymä on 30,62.

Ja seuraava rivi:  hav. 10m korkeudella,  taajuus 1,00x10^18Hz,  punasiirtymä 98,83 ?

jussi_k_kojootti

Lainaus käyttäjältä: mistral - 10.03.2017, 19:05:59
Luetaanko ylin rivi näin: havaitsija on 1m korkeudella ja mittaa horisontista tulevan taajuuden 3,16x10^18Hz. Punasiirtymä on 30,62.
Ja seuraava rivi:  hav. 10m korkeudella,  taajuus 1,00x10^18Hz,  punasiirtymä 98,83 ?

Tismalleen.  Ylemmässä taulukossa havaitsija pysyy paikallaan (oletus: 1au etäisyydellä), ja emitteri siirtyy kaukaa (>1au etäisyydeltä) kohti horisonttia.

Laskuri ei salli emitterin tai havaitsijan olla horisontin sisäpuolella (käytännössä antaa vääriä tuloksia, tai ei toimi ollenkaan).
jussi kantola / oulun arktos
CG-5 GOTO + KWIQ-guiding + SW80ED  // 10" dobson // canon eos 450d mod & 400d / ASI 120MM
http://astrobin.com/users/jussi_k_kojootti/
http://oulunarktos.fi/

pnuu

Lainaus käyttäjältä: Joksa - 10.03.2017, 19:04:57
Arvio tuli siitä että energisimmänkin fotonin pitää pysähtyä tasan Killingin horisontin rajapintaan, ei siis riitä että pysähtyisi jonnekin g-kenttään, ja ei ole löytynyt mitään viitettä siitä että fotonin energialla olisi joku äärellinen raja-arvo.

Valo kulkee suoraan. Avaruus kaareutuu mustanaukon lähellä. Valo kulkee suoraan mustanaukon ympärille kaareutuvaa avaruutta pitkin. Suljettua pintaa pitkin kulkiessa ei ole muuta suoraa reittiä, joten ulos ei pääse.

Unohtakaa nyt jo se rakettianalogia...

Panu Lahtinen
"You haven't really been anywhere until you've got back home",
Twoflower in "The Light Fantastic"

Eusa

Lainaus käyttäjältä: ketarax - 10.03.2017, 20:36:10
Tismalleen.  Ylemmässä taulukossa havaitsija pysyy paikallaan (oletus: 1au etäisyydellä), ja emitteri siirtyy kaukaa (>1au etäisyydeltä) kohti horisonttia.

Laskuri ei salli emitterin tai havaitsijan olla horisontin sisäpuolella (käytännössä antaa vääriä tuloksia, tai ei toimi ollenkaan).
Horisontin sisäpuolelta ei voi saada määritelmällisesti mitään informaatiota ulos, joten ohjelmasi toimii siten mainiosti.

mistral

Lainaus käyttäjältä: pnuu - 10.03.2017, 20:45:17
Valo kulkee suoraan. Avaruus kaareutuu mustanaukon lähellä. Valo kulkee suoraan mustanaukon ympärille kaareutuvaa avaruutta pitkin. Suljettua pintaa pitkin kulkiessa ei ole muuta suoraa reittiä, joten ulos ei pääse.

Unohtakaa nyt jo se rakettianalogia...

Joo unohdetaan vaan mutta huomenna se taas alkaa kummitella. Ongelma on ma:n massakeskipiste joka ei saisi tuottaa äkkijyrkkää kenttää vaan jouhevan muodon g-kaivolle. Tässä horisontti on äkkijyrkkä ikäänkuin "lasikatto". Sinänsä kaava

                     R1 (R2 - Rs)
  f1 = f2 sqrt[ -------------- ]
                     R2 (R1 - Rs)

on onnistuttu sievistämään yksinkertaiseksi mikä tekee sen uskottavaksi mutta se äkkijyrkkyys hämää.

Wikistä löytynyt Killingin horisontti vaikuttaa jouhevammalta eli siinä g-voimat on newtonilaisesti ymmärrettävissä.


jussi_k_kojootti

Lainaus käyttäjältä: Eusa - 11.03.2017, 00:06:26
Horisontin sisäpuolelta ei voi saada määritelmällisesti mitään informaatiota ulos, joten ohjelmasi toimii siten mainiosti.

Kyllä kyllä, mutta input-parametrien tarkisteluun jäi varmaan puutteita, ja/tai syötettyjä arvoja pakotetaan fysikaalisiin rajoihin sen kummemmin kommentoimatta.
jussi kantola / oulun arktos
CG-5 GOTO + KWIQ-guiding + SW80ED  // 10" dobson // canon eos 450d mod & 400d / ASI 120MM
http://astrobin.com/users/jussi_k_kojootti/
http://oulunarktos.fi/